Форум » Теория » Критерий выбора рабочей точки » Ответить

Критерий выбора рабочей точки

B_Aleko: Приветствую всех. Решил заняться сборкой ламповых усилителей вплотную. Ранее, лет 10-12 назад, был небольшой опыт сборки 5 усилков..., в подробности их работы, особо не вникал, но солидные люди интересовались, слушали... и продал. Но несколько месяцев всё прочитываю литературу, и никак не могу выяснить способ выбора рабочей точки "с нуля", то есть: взять лампу, её ВАХ и... поставить точку РТ. И меня смущает тот факт, что во многих пересмотренных схемах, РТ всегда располагается ниже тока, который заявлен в параметрах ламп (подробнее объясню в обсуждении). Это еще не все. Будет масса интересных "моих" вычислений по этой теме и другим. Прошу подробно объяснить данное вычисление или принцип, отнестись без скептицизма и как к новичку. Спасибо.

Ответов - 246, стр: 1 2 3 4 5 6 7 8 9 10 11 12 13 All

B_Aleko: Пермяк пишет: на частоте 6,3 Гц синусоидальный сигнал уменьшится на 3 дБ. хорошо, но если мне нужен спад ниже 40 Гц? для уменьшения габаритов выходного транса.

ALSS: В случае сужения полосы сигнала снизу для уменьшения габаритов выходного трансформатора просто подставляется другая частота в расчеты и обязательно вводится ограничение полосы перед/между каскадами, чтобы на этот уменьшенный трансформатор не пришли частоты ниже его рабочего диапазона и не ввели его сердечник в насыщение. Таким образом пытаются делать усилители для мидбаса или для ШП с поддержкой снизу. Ну и ПРО-системы озвучивания. Вопросы задержек/опережения сигнала в полосах трогать не будем. Конечно же следует учитывать эффект накопления ограничений, о котором пишет Пермяк, вследствие увеличения порядка фильтра. Для озвучивания этот эффект полезен. Однако тут вступает в действие субъективный эффект - чем сильнее ограничена полоса усилителя снизу, тем мельче масштаб воспроизводимой музыки. Для музыки лучше ограничивать - по напряжению! Не по полосе - входной сигнал и работать на меньшей мощности, но в максимально широкой полосе. О том, что это мое субъективное мнение, напоминать далее не стОит? Пермяк пишет: меандр частотой 40 Гц, то спад вершин прямоугольных импульсов меандра не превысит 10% Нет, к сожалению, для 10% спада вершины на частоте меандра 40 Гц нижняя частота д. б. гораздо ниже, 0,16*0,1/0,0125=1,28 Гц. Впрочем, в данном случае это мелочи второго порядка малости.

Пермяк: Да, прошу прощения, ошибся: при Fн(-3дБ)=6,3 Гц скол импульса меандра частотой 40 Гц равен 50%.


Пермяк: B_Aleko пишет: мне нужен спад ниже 40 Гц... для уменьшения габаритов выходного транса.Коллега, если бы Вы сразу задали вопрос именно так, то сразу получили бы конкретный ответ: для этой цели Ск как правило не применяется. Предположим, что мы постепенно уменьшаем частоту синус-сигнала от СЧ до НЧ, и следим за уровнем на выходе каскада. Можно подобрать ёмкость Ск так, что на какой-то частоте, скажем 30 Гц, спад будет равен 3дБ. Но если мы и дальше будем уменьшать частоту, то с некоторого момента увидим, что спад постепенно уменьшается, и на какой-то частоте спада вообще не будет. Просто потому, что на этой частоте ёмкостное сопротивление конденсатора Ск уже очень большое, он практически не шунтирует Rк, и усиление сигнала каскадом (хотя и охваченным ООС) сохраняется вплоть до 0 Гц. Примеры вида АЧХ на НЧ при наличии Ск. Начало спада и его прекращение зависят от исходных параметров лампы и соотношения величин Rа и Rк, и зависимость эта достаточно сложная. Вывод: ожидать от Ск эффективной работы в качестве фильтра не следует.

B_Aleko: Пермяк пишет: получили бы конкретный ответ: как-то в начале не подумал о трансе. тогда: при спаде усиления, к примеру в 3 дб., в драйвере, можно настроить последующую цепочку из Rc и Cp тоже со спадом аналогичной частоты. и получим уже в 6 дб.?

B_Aleko: при Rc=220 кОм и Cp=22нФ, спад при частоте 32,88 Гц. если правильно, тогда на сколько дб?

Пермяк: На пост 165: На этой частоте суммарный спад, обусловленный этими двумя цепочками, будет 6 дБ. Но на более низких частотах крутизна спада уменьшится.

B_Aleko: то есть, уровень на частоте снизится и останется без изменений вплоть до 0Гц?

Пермяк: Нет. Цепочка CрRc по-прежнему будет давать спад, и при 0 Гц (постоянный ток!) не пропустит ничего :) B_Aleko пишет: при Rc=220 кОм и Cp=22нФ, спад при частоте 32,88 Гц. если правильно, тогда на сколько дб?Если имеете в виду только эту цепочку, то о ней всё сказано здесь: http://hiend.borda.ru/?1-13-0-00000093-000-0-0-1453553729

B_Aleko: Добрый день. У меня возникло сомнение о величине: Внутреннее сопротивление Ri - сопротивление лампы переменному току. Определяется как отношение изменения анодного напряжения к изменению анодного тока при неизменных напряжениях на остальных электродах. Измеряется в кОм. Внутреннее сопротивление может быть определено по значениям крутизны характеристики S и коэффициента усиления M: Ri=M/S Тогда чем отличается сопротивление постоянному току от сопротивления по переменному току?

Пермяк: B_Aleko Здравствуйте! Вы задали очень интересный вопрос. Допускаю, что с математикой у Вас не очень, поэтому попробую на этот теоретический вопрос ответить доступным (как мне кажется) языком. Во-первых, неверны сами по себе выложенные Вами определения. Вот как надо: 1. Сопротивление лампы постоянному току - это сопротивление в заданной (или выбранной) точке на семействе статических! ВАХ лампы. Определяется по закону Ома для пост. тока для неизменных Ua и Ia .Этому сопротивлению даже буквы специальной не присвоено, настолько оно не нужно при расчётах. 2. Ri - это сопротивление, проявлямое лампой в динамическом режиме, т.е. - при подключенной к ней нагрузке и изменении Ua и Ia при движении рабочей точки по нагрузочной линии (в нашем случае - по прямой Ra). Образно можно описать процесс так. Под воздействием изменения Uc рабочая точка пересекает линии ВАХ, "наталкиваясь на них поочерёдно. При этом ей кажется, что она движется по плоскости, сплошь заполненной бесконечным множеством линий ВАХ, которые имеют примерно одинаковый наклон. И переменный ток через лампу "видит" сопротивление, которое равно dU/dI. Что такое это самое dU/dI, Вы уже знаете: оно и есть Ri. Если моё пояснение что-то просветило... А если нет - то придётся углубляться в математику. Посмотрим.

Пермяк: to B_Aleko Я удалил из темы весь ранее следовавший за предыдущим постом флейм. Вы проводите какие-то исследования, касающиеся параметров лампы? Хорошо. Когда закончите эту работу, милости просим сюда, выложить результаты. Желательно на понятном русском языке, чтобы не пришлось Вас переспрашивать.

B_Aleko: хорошо. согласен. Только моя просьба остается в силе. конфиденциально, в личку контакты.

B_Aleko: Хотелось бы вернуться к вопросу о величине Ri. Это же тема о теории. Пермяк пишет:1. Сопротивление лампы постоянному току - это сопротивление в заданной (или выбранной) точке на семействе статических! ВАХ лампы.Это то же сопротивление лампы? но как быть с физикой? ведь не существует разных сопротивлений для двух величин (под переменный и постоянный ток). тем более в одном баллоне.

Ученик: B_Aleko, Вам уже ответил Пермяк:Этому сопротивлению даже буквы специальной не присвоено, настолько оно не нужно при расчётах. B_Aleko пишет: но как быть с физикой? Учить.ведь не существует разных сопротивлений для двух величин (под переменный и постоянный ток). Ещё как существует.

Пермяк: Попытаюсь ещё раз. Подключим к выводам накала лампы полагающееся ей по паспорту напряжение. Подключим между сеткой и катодом лампы источник напряжения 3В "минусом" к сетке. Между катодом и анодом подключим регулируемый источник напряжения "минусом" к катоду, и установим на его выходе напряжение 150 вольт. Согласно графику семейства ВАХ ток через ламу будет равен 5 мА. Если взять отношение абсолютных значений напряжения и тока, получим сопротивление пост. току: R=U/I=150/5=30 кОм. Проведём прямую АВ. Отметим, что для любой точки на этой прямой вычисленное R всегда будет равно 30 кОм. Т.е., получили сопротивление лампы постоянному току. Куда применить при вычислениях - неизвестно. Увеличим напряжение анодного источника до 165 вольт. Т.е., дадим приращение ∆Ua=165-150=15 В Под воздействием приращения напряжения произойдёт приращение анодного тока с 5 до 7 мА: ∆Ia=7-5=2 мА Если возьмём отношение этих относительных приращений, получим величину: Ri=∆Ua/∆Ia =15/2= 7,5 В/мА. Поскольку В/мА=кОм, то есть Ri выражается в единицах сопротивления, то величине Ri дали название ВНУТРЕННЕЕ СОПРОТИВЛЕНИЕ лампы. В нашем случае, для точки А: Ri=7,5 кОм. Таким образом имеем два похожих названия для разных величин, сильно отличающихся одна от другой как по сути, так и в числовом выражении. ЗЫ. Кстати, описанным под рисунком методом и измеряют Ri. ЗЫ.ЗЫ. Поскольку ВАХа лампы не является прямой линией, то, РАЗУМЕЕТСЯ, что в разных её точках величина Ri разная: чем ниже на линии ВАХ находится точка, тем Ri - выше. Всё семейство ВАХов для конкретной лампы состоит из математически неправильных кривых, и чем кривая ближе к оси Ia, тем меньше Ri в её точках при одном и том же значении Ia.

B_Aleko: Пермяк пишет: Попытаюсь ещё раз. Да, чисто теоретически, согласен. Можно ли где, найти первоначальный источник данного решения? И, по какому закону "сопротивление, проявлямое лампой в динамическом режиме", это движение рабочей точки по нагрузочной линии осуществляется? http://akadem-nauki.ru/dinamicheskij-rezhim-raboty-bpt-dinamicheskaya-xarakteristika/ только это такое же решение построенное по одним вычислениям, а сам закон?

Пермяк: Представим себе, что лампа полностью заперта. Ток через неё равен нулю. Тогда равен нулю и ток через Ra. Падения напряжения на Ra нет, и всё напряжение питания Ea приложено к аноду. Имеем точку с координатами (Ua=Ea; Ia=0), она лежит на оси напряжений. Теперь представим себе, что лампа полностью открыта. Через неё идёт ток теоретически максимальный, ограниченный только сопротивлением Ra: Ia.мах=Еа/Ra. Имеем вторую точку, с координатами (Ua=0; Ia=Ia.maх). Соединяем полученные две точки прямой линией (нагрузка активная), получаем траекторию перемещения рабочей точки. Таким образом, раб. точка перемещается по линейному ЗАКОНУ... э-э ... в данной системе координат

B_Aleko: "Теперь представим себе, что лампа полностью открыта....", по теоретическому вычислению, да. но практически этого никогда нет. линия Ra должна иметь совокупность и пропорцию Амплитуда-время. вот на графике видно, что красная линия нагрузки это совокупность и пропорция Амплитуда-время. А применение величины Ri по отрезку А-С, вносит искажение по времени.

Пермяк: B_Aleko пишет: Это справедливо при нулевом потенциале сетки и катода. И в этом случае, Ri = (Ua=Ea)/(Ia=0) =290k. Если сразу задать смещение, то этих величин не будет Ну что Вы такое пишете! Мы строим нагр. прямую для переменного тока, а катод соединён с землёй накоротко по перем. току через конденсатор Ск. Падающее на Rк ПОСТОЯННОЕ напряжение в построении не участвует, просто надо после всех вычислений добавить к Еа падающее на Rк постоянное напряжение.



полная версия страницы